Antiderivative of a rational function

In summary: You have the freedom to add a linear term to the last partial fraction. The linear term is not forbidden by the denominator, it is the same as if the numerator of the rational function contained it.
  • #1
nuuskur
Science Advisor
880
938

Homework Statement


Consider the integral:
[tex]\int\frac{2x^3 -4x^2 +8x +7}{(x-1)^2 (x^2 +4x +8)}{\rm{d}}x[/tex]

Homework Equations

The Attempt at a Solution


The degree of the denominator is 4 and the numerator's is 3, hence I thought I would try partial fractions:
[tex]\frac{A}{x-1} +\frac{B}{(x-1)^2} +\frac{C}{x^2 +4x +8} = \frac{2x^3 -4x^2 +8x +7}{(x-1)^2 (x^2 +4x +8)}[/tex]multiplying both sides by the denominator on the right side we would have:
[tex]A(x-1)(x^2 +4x +8) +B(x^2 +4x +8) +C(x-1)^2 = 2x^3 -4x^2 +8x +7\\Ax^3 +(3A +B +C)x^2 +(4A +4B -2C)x -(8A -8B -C) = 2x^3 -4x^2 +8x +7[/tex]
So I should be able to conclude that A = 2, however, the problem is that on one hand I get that 3B = -10 and on the other hand, 10B = 23. Have I made a mistake in the calculations? Is any such rational function divisible [not sure if that's the correct word] into partial fractions?
Is there any other method for tackling such a problem?

Thank you in advance.
 
Last edited:
Physics news on Phys.org
  • #2
nuuskur said:

Homework Statement


Consider the integral:
[tex]\int\frac{2x^3 -4x^2 +8x +7}{(x-1)^2 (x^2 +4x +8)}{\rm{d}}x[/tex]

Homework Equations

The Attempt at a Solution


The degree of the denominator is 4 and the numerator's is 3, hence I thought I would try partial fractions:
[tex]\frac{A}{x-1} +\frac{B}{(x-1)^2} +\frac{C}{x^2 +4x +8} = \frac{2x^3 -4x^2 +8x +7}{(x-1)^2 (x^2 +4x +8)}[/tex]multiplying both sides by the denominator on the right side we would have:
[tex]A(x-1)(x^2 +4x +8) +B(x^2 +4x +8) +C(x-1)^2 = 2x^3 -4x^2 +8x +7\\Ax^3 +(3A +B +C)x^2 +(4A +4B -2C)x -(8A -8B -C) = 2x^3 -4x^2 +8x +7[/tex]
So I should be able to conclude that A = 2, however, the problem is that on one hand I get that 3B = -10 and on the other hand, 10B = 23. Have I made a mistake in the calculations? Is any such rational function divisible [not sure if that's the correct word] into partial fractions?
Is there any other method for tackling such a problem?

Thank you in advance.
The last partial fraction should contain an additional first-order term in the numerator: [tex]\frac{A}{x-1} +\frac{B}{(x-1)^2} +\frac{Cx+D}{x^2 +4x +8} = \frac{2x^3 -4x^2 +8x +7}{(x-1)^2 (x^2 +4x +8)}[/tex]
 
  • #3
Thank you, ehild, for the correction. Could you please explain why the last partial fraction should also contain the first-order term? I do not doubt your words, I can't fully understand the concept myself.
Everything works out nicely, though.
A = 0, B = 1, C = 2 and D = -1 and the rest is trivial.
 
  • #4
The denominator is of order 2, and you always need one order less in the numerator (for your (x-1)-type fractions, the A serves that purpose).

As a simple example, you cannot express ##\frac{5x+3}{x^2+4x+8}## with ##\frac{C}{x^2+4x+8}##.
 
  • Like
Likes nuuskur
  • #5
nuuskur said:
Could you please explain why the last partial fraction should also contain the first-order term? I do not doubt your words, I can't fully understand the concept myself.
As you have a third-order polynomial in the numerator, it involves 4 equations when comparing its four coefficients with the expression obtained from the partial fractions. So you need 4 unknowns in general. Otherwise you might arrive at contradiction, as you experienced.
 
  • Like
Likes nuuskur

Related to Antiderivative of a rational function

1. What is an antiderivative?

An antiderivative is the inverse operation of a derivative. It is a function that, when differentiated, gives the original function. It is also known as the indefinite integral.

2. What is a rational function?

A rational function is a function that can be written as a ratio of two polynomials, where the denominator is not equal to zero. It is also known as a fraction function.

3. How do you find the antiderivative of a rational function?

To find the antiderivative of a rational function, you can use the power rule for integration, where the power of the variable is increased by one and divided by the new power. You can also use the substitution method or partial fraction decomposition, depending on the complexity of the function.

4. What is the difference between a definite and indefinite antiderivative?

A definite antiderivative has specific limits of integration, while an indefinite antiderivative does not. A definite antiderivative gives a numerical value, while an indefinite antiderivative is expressed as a function.

5. Can a rational function have more than one antiderivative?

Yes, a rational function can have multiple antiderivatives, as adding a constant term to the antiderivative does not change its derivative. This is known as the constant of integration.

Similar threads

  • Calculus and Beyond Homework Help
Replies
3
Views
624
  • Calculus and Beyond Homework Help
Replies
10
Views
528
  • Calculus and Beyond Homework Help
Replies
5
Views
804
  • Calculus and Beyond Homework Help
Replies
9
Views
1K
  • Calculus and Beyond Homework Help
Replies
9
Views
2K
  • Calculus and Beyond Homework Help
Replies
8
Views
909
  • Calculus and Beyond Homework Help
Replies
2
Views
583
  • Calculus and Beyond Homework Help
Replies
3
Views
847
  • Calculus and Beyond Homework Help
Replies
4
Views
934
  • Calculus and Beyond Homework Help
Replies
25
Views
461
Back
Top